Vector with the same direction but a different magnitude

Click For Summary
To find a vector with the same direction as <-2, 4, 2> but a length of 6, the original vector's length is calculated as 24. The correct scaling factor to achieve the desired length is determined to be 1.5, but the initial application of this factor was incorrect, leading to confusion. The error was in the notation used for scaling the vector components. Ultimately, the correct approach involves properly applying the scaling factor to the vector's components to ensure the resultant vector has the desired magnitude.
Calpalned
Messages
297
Reaction score
6

Homework Statement


Find a vector that has the same direction as <-2, 4, 2> but has length 6.

Homework Equations


Length of vector in 3D = |v| = (x2 + y2 + z2)0.5

The Attempt at a Solution


I can see that the length of the given vector is (-2)2 + 42 + 22 = 24
36 is (1.5)24, so I tried multiplying each component of the vector by the scalar 1.5. However, this doesn't give me the desired length of six (rather, it gives me the square root of 54). I had a feeling this method would be faulty, but nothing else comes to mind.
 
Physics news on Phys.org
Compare lengths not the squares of lengths to get the right factor.
 
Calpalned said:
I can see that the length of the given vector is (-2)2 + 42 + 22 = 24
Not according to the formula you included under "relevant equations".
 
Calpalned said:

Homework Statement


Find a vector that has the same direction as <-2, 4, 2> but has length 6.

Homework Equations


Length of vector in 3D = |v| = (x2 + y2 + z2)0.5

The Attempt at a Solution


I can see that the length of the given vector is (-2)2 + 42 + 22 = 24
36 is (1.5)24, so I tried multiplying each component of the vector by the scalar 1.5. However, this doesn't give me the desired length of six (rather, it gives me the square root of 54). I had a feeling this method would be faulty, but nothing else comes to mind.

I tried it again with the length of the desired vector = (F(-2)2 + F42 + F22)0.5 Where F = 1.5, and the answer seems to be correct...
 
I just noticed my error. In my original attempt, (F * IJK)2 for each component I did when it should have been F(IJK2)
 
Calpalned said:
I just noticed my error. In my original attempt, (F * IJK)2 for each component I did when it should have been F(IJK2)
Perhaps you understand what you mean, but this is very weird notation. If u = <a, b, c> and v = 2<a, b, c>, then |v| = 2|u|
 
Question: A clock's minute hand has length 4 and its hour hand has length 3. What is the distance between the tips at the moment when it is increasing most rapidly?(Putnam Exam Question) Answer: Making assumption that both the hands moves at constant angular velocities, the answer is ## \sqrt{7} .## But don't you think this assumption is somewhat doubtful and wrong?

Similar threads

  • · Replies 14 ·
Replies
14
Views
2K
  • · Replies 10 ·
Replies
10
Views
5K
  • · Replies 2 ·
Replies
2
Views
5K
  • · Replies 1 ·
Replies
1
Views
3K
  • · Replies 9 ·
Replies
9
Views
8K
  • · Replies 1 ·
Replies
1
Views
2K
  • · Replies 9 ·
Replies
9
Views
1K
Replies
2
Views
2K
Replies
5
Views
2K
  • · Replies 2 ·
Replies
2
Views
1K